SchoolWork-LaTeX/随机过程/作业/第二周作业.tex

187 lines
11 KiB
TeX
Raw Blame History

This file contains ambiguous Unicode characters

This file contains Unicode characters that might be confused with other characters. If you think that this is intentional, you can safely ignore this warning. Use the Escape button to reveal them.

\documentclass[全部作业]{subfiles}
\input{mysubpreamble}
\begin{document}
\setcounter{chapter}{1}
\section{《随机过程》第二周作业}
\setcounter{subsection}{2}
\subsection{练习题\thechapter.\thesubsection}
\begin{enumerate}
\questionandanswerProof[1]{
$X$是连续型非负随机变量,证明$L_{X}(t)$$[0,+\infty)$内是一致连续的。
}{
% 注意到$f(t)=e^{-t}$在$[0,+\infty)$内是一致连续的,即$\forall \varepsilon>0, \exists \delta_0 >0, \forall t_1,t_2 \in [0,+\infty)$,且$\left\vert t_1-t_2 \right\vert <\delta_0$,都有$\left\vert e^{-t_1}-e^{-t_2} \right\vert <\dfrac{\varepsilon}{Ee^{X}}$,所以$Ee^{X}\left\vert e^{-t_1}-e^{-t_2} \right\vert <\varepsilon$,即$\left\vert Ee^{-t_1X}-Ee^{-t_2X} \right\vert =Ee^{-t_1X}(1-e^{(t_1-t_2)X})$
$X$的概率密度函数为$p(x)$。使用点列式等价定义,$\forall t_{1n}, t_{2n}\in [0,+\infty)$$\left\vert t_{1n}-t_{2n} \right\vert \to 0$
$$
\begin{aligned}
&\left\vert L_{X}(t_{1n})-L_{X}(t_{2n}) \right\vert = \left\vert Ee^{-t_{1n}X}-Ee^{-t_{2n}X} \right\vert =\left\vert E(e^{-t_{1n}X}-e^{-t_{2n}X}) \right\vert \\
&=\left\vert \int_{0}^{+\infty} (e^{-t_{1n}x}-e^{-t_{2n}x})p(x) \mathrm{d}x \right\vert =\left\vert \int_{0}^{+\infty} e^{-t_{1n}x}(1-e^{(t_{1n}-t_{2n})x}) p(x)\mathrm{d}x \right\vert \\
&\leqslant \left\vert \int_{0}^{+\infty} (1-e^{(t_{1n}-t_{2n})x}) p(x)\mathrm{d}x \right\vert \\
\end{aligned}
$$
根据积分中值定理,$\exists \xi \in [0,+\infty) $,使得
$$
\left\vert \int_{0}^{+\infty} (1-e^{(t_{1n}-t_{2n})x})p(x) \mathrm{d}x \right\vert =(1-e^{(t_{1n}-t_{2n})\xi}) \int_{0}^{+\infty} p(x) \mathrm{d}x=1-e^{(t_{1n}-t_{2n})x}
$$
因为$\left\vert t_{1n}-t_{2n} \right\vert \to 0$,所以$1-e^{(t_{1n}-t_{2n})x} \to 0$,因此$\left\vert L_{X}(t_{1n}-t_{2n}) \right\vert \to 0$,从而$L_{X}(t)$$[0,+\infty)$内是一致收敛的。
}
\questionandanswer[5]{
设独立同分布的标准正态分布随机变量簇$W_m,m>1$$\{ N(n),n\geqslant 1 \}$独立,其中$N(n)$服从泊松分布$P(n)$。令$Y_n=\sum_{k=1}^{N(n)}W_k$。求$Y_n$的特征函数,并证明$n \to +\infty$$\dfrac{Y_n}{\sqrt{n}}$依分布收敛到一个服从标准正态分布的随机变量。
}{
\begin{solution}
$Y_n$的特征函数为
$$
\begin{aligned}
\psi_{Y_n}(t)=E e^{it Y_n}=Ee^{it \sum_{k=1}^{N(n)}W_k}=E e^{\sum_{k=1}^{N(n)}it W_k}=E \prod_{k=1}^{N(n)} e^{it W_k}
\end{aligned}
$$
根据重期望公式,
$$
\text{上式}=E\left( E \left(\left. \prod_{k=1}^{N(n)} e^{it W_k} \right| N(n)=p \right) \right)
$$
由于$W_k$独立同分布且服从标准正态分布,所以
$$
E\left( \left. \prod_{k=1}^{N(n)} e^{it W_k} \right|N(n)=p \right) =\prod_{k=1}^{p} E e^{it W_k}= \prod_{k=1}^{p} \psi_{W_1}(t)= \left[ \psi_{W_1}(t) \right] ^{p}=\left[ e^{-\frac{t^{2}}{2}} \right] ^{p}=e^{-\frac{p t^{2}}{2}}
$$
所以
% = e^{- n} \sum_{k=1}^{\infty} \frac{n^{k} e^{- \frac{k t^{2}}{2}}}{\Gamma(k + 1)}
$$
\psi_{Y_n}(t)=E \left( e^{-\frac{t^{2}N(n)}{2}} \right) = \sum_{k=1}^{\infty}e^{- \frac{t^{2}k}{2}} \frac{n^{k}}{k!}e^{-n}
% = e^{- \frac{nt^{2}}{2}}
$$
\end{solution}
\begin{proof}
根据特征函数的性质,
$$
\psi_{\frac{Y_n}{\sqrt{n}}}(t)=\psi_{Y_n}\left( \frac{t}{\sqrt{n}} \right)
% =e^{-\frac{n \left( \frac{t}{\sqrt{n}} \right) ^{2}}{2}}=e^{-\frac{t^{2}}{2}}
= \sum_{k=1}^{\infty}e^{-\frac{\left( \frac{t}{\sqrt{n}} \right) ^{2} \cdot k}{2}} \frac{n^{k}}{k!}e^{-n} = e^{- n} \sum_{k=1}^{\infty} \frac{n^{k} e^{- \frac{k t^{2}}{2 n}}}{k!}
$$
$n \to +\infty$时上式$\to e^{-\frac{t^{2}}{2}}$,即$\frac{Y_n}{\sqrt{n}}$的特征函数收敛到标准正态分布的特征函数。根据特征函数的唯一性,则$n \to +\infty$$\frac{Y_n}{\sqrt{n}}$依分布收敛到一个服从标准正态分布的随机变量。
\end{proof}
}
\questionandanswerSolution[7]{
若随机变量$X$的分布列为$P(X=n)=\frac{n}{2^{n+1}},n\geqslant 1$,求$X$的概率母函数$\phi(s)$
}{
$$
\phi(s)=E s^{X}=\sum_{n=1}^{\infty}s^{n} \cdot \frac{n}{2^{n+1}} = \frac{s}{(s - 2)^{2}}\ , \quad s \in [-1,1]
$$
}
\questionandanswerSolution[8]{
已知概率母函数$\phi(s)=s+ \frac{1}{1+\gamma}(1-s)^{1+\gamma}, \gamma\in (0,1]$,求对应的概率分布列。
}{
$$
P(X=k)=\frac{\phi^{(k)}(0)}{k!}
$$
\begin{minipage}{0.6\linewidth}
$\phi(s)$求各阶导数:
$$
\phi^{(k)}(x)=\begin{cases}
s+ \frac{1}{1+\gamma}(1-s)^{1+\gamma},\quad & k=0 \\
1-(1-s)^{\gamma},\quad & k=1 \\
\displaystyle (-1)^{k}(1-s)^{\gamma-k+1} \prod_{i=0}^{k-2} (\gamma-i),\quad & k\geqslant 2 \\
\end{cases};
$$
\end{minipage}
\begin{minipage}{0.4\linewidth}
所以
$$
\phi^{(k)}(0)=\begin{cases}
\frac{1}{1+\gamma},\quad & k=0 \\
0,\quad & k=1 \\
\displaystyle (-1)^{k}\prod_{i=0}^{k-2} (\gamma-i),\quad & k\geqslant 2 \\
\end{cases}
$$
\end{minipage}
所以对应的概率分布列为
$$
P(X=k)=\begin{cases}
\frac{1}{1+\gamma},\quad & k=0 \\
0,\quad & k=1 \\
\displaystyle \frac{(-1)^{k}}{k!}\prod_{i=0}^{k-2}(\gamma-i) ,\quad & k\geqslant 2 \\
\end{cases}
$$
}
\end{enumerate}
\pagebreak[4] % 不知道为什么这行如果强度改成4会导致页码问题
\subsection{练习题\thechapter.\thesubsection}
\begin{enumerate}
\questionandanswerProof[1]{
若存在随机变量$X$使得随机序列$\dfrac{S_n}{n}\xlongrightarrow{a.s.}X$成立,而且非负整数值随机序列$N_n \xlongrightarrow{a.s.}\infty$成立。证明$\dfrac{S_{N_n}}{N_n}\xlongrightarrow{a.s.}X$
}{
对于$\forall \varepsilon>0$,由$\frac{S_n}{n} \xlongrightarrow{a.s.} X$的等价定义,$\exists A \in \Omega$满足$P(A)=0$,对$\forall \omega \in \Omega A$以及$\forall \varepsilon>0$$\exists N=N(\varepsilon,\omega)$,使得当$n>N$$\left\vert \frac{S_n(\omega)}{n}-X(\omega) \right\vert <\varepsilon$
对于上述的$N$,由$N_n\xlongrightarrow{a.s.}\infty$的等价定义,$\exists A'\in \Omega$满足$P(A')=0$,对$\forall \omega' \in \Omega A$$\exists M=M(\varepsilon,\omega)$,使得当$n>M$$N_n(\omega')> N$,再由上述叙述可知$\left\vert \displaystyle \frac{S_{N_n(\omega')}(\omega)}{N_n(\omega')}-X(\omega) \right\vert <\varepsilon$
因此$\displaystyle \frac{S_{N_n}}{N_n}\xlongrightarrow{a.s.}X$
}
\questionandanswer[2]{
证明:若对任意$\varepsilon>0$,存在$N$使得当$ n>N$$P(\left\vert X_n-X \right\vert <\varepsilon)=1$,那么$X_n\xlongrightarrow{a.s.}X$。举例说明逆命题不成立。
}{
\begin{proof}
由于$P(\left\vert X_n-X \right\vert <\varepsilon)=1 \Rightarrow P(\left\vert X_n-X \right\vert \geqslant \varepsilon)=0$
题中条件可转换为
$$
\text{}\forall \varepsilon>0, P\left( \bigcap_{N=1} ^{\infty}\bigcup_{n=N} ^{\infty}\{ \left\vert X_n-X \right\vert \geqslant \varepsilon \}\right)=0
$$
所以$X_n\xlongrightarrow{a.s.}X$
\end{proof}
$P(\left\vert X_n-X \right\vert \geqslant \varepsilon)=0$$P(\left\vert X_n-X \right\vert <\varepsilon)\neq 1$时,逆命题不成立。
}
\questionandanswerProof[4]{
$f(x)$$[0,+\infty)$上连续有界,单调上升且$f(0)=0$,证明随机变量$X_n$依概率收敛于0当且仅当$\displaystyle \lim_{n \to \infty}E(f(\left\vert X_n \right\vert ))=0$
}{
$X_n \xrightarrow{P} 0$,则$\forall \varepsilon>0, P(\left\vert X_n \right\vert >\varepsilon) \to 0$,由于$f(x)$的单调性,$P(f(\left\vert X_n \right\vert )>f(\varepsilon)) \to 0$
$\varepsilon \to 0$,则$f(\varepsilon)\to f(0)=0$,所以$P(f(\left\vert X_n \right\vert )>0)\to 0$
因为$\left\vert X_n \right\vert \geqslant 0$,且$f(x)$单调上升且$f(0)=0$,所以$f(\left\vert X_n \right\vert )\geqslant 0$,因此$P(f(\left\vert X_n \right\vert )=0)\to 1$
$f(\left\vert X_n \right\vert )$在0处的概率趋向于1所以$\displaystyle \lim_{n \to \infty}E(f\left\vert X_n \right\vert )=0$
另一方向:
$\displaystyle \lim_{n \to \infty}E(f(\left\vert X_n \right\vert ))=0$,由于$f(\left\vert X_n \right\vert )$是非负随机变量所以期望为0等价于在0处的概率为1$\displaystyle \lim_{n \to \infty}P(f(\left\vert X_n \right\vert )=0)=1$,即$P(f(\left\vert X_n \right\vert )=0)\to 1$
由于$f(\left\vert X_n \right\vert )\geqslant 0$,所以$P(f(\left\vert X_n \right\vert )>0)\to 0$
再根据$f(x)$的单调性,$P(\left\vert X_n \right\vert >0) \to 0$
因此$\forall \varepsilon>0, P(\left\vert X_n \right\vert >\varepsilon) \to 0$,即$X_n$依概率收敛于0。
}
\questionandanswerProof[5]{
$\{ X_k;k\geqslant 1 \}$为一列相互独立随机变量,而且对任意$k\geqslant 1$$E(X_k)=0$$\operatorname{Var}(X_k)=k$。对任意$n\geqslant 1$,令$\displaystyle Y_n=\sum_{k=1}^{n}\frac{X_k}{k}$
证明对任意$r>\dfrac{1}{2}$,当$n \to \infty$$\dfrac{Y_n}{n^{r}}$几乎必然收敛到0。
}{
要证$\dfrac{Y_n}{n^{r}}\xrightarrow{a.s.}0$,即证$\forall \varepsilon>0, \displaystyle \sum_{n=1}^{\infty}P\left( \left\vert \frac{Y_n}{n^{r}}\right\vert>\varepsilon \right) <\infty $
由马尔可夫不等式,
$$
\displaystyle P\left( \left\vert \frac{Y_n}{n^{r}} \right\vert >\varepsilon \right) <\frac{E\left\vert \frac{Y_n}{n^{r}} \right\vert }{\varepsilon}=\frac{E\left\vert \displaystyle \sum_{k=1}^{n}\frac{X_k}{k} \right\vert }{n^{r}\varepsilon}
$$
由于$\operatorname{Var}(X_k)=k, std(X_k)=\sqrt{k}$$E(X_k)=0$,所以$std(\frac{X_k}{k})=\frac{1}{\sqrt{k}}$,所以$\operatorname{Var}(\frac{X_k}{k})=\frac{1}{k}$
$$
E\left\vert \sum_{k=1}^{n}\frac{X_k}{k} \right\vert \leqslant \left\vert \sum_{k=1}^{n}\frac{1}{\sqrt{k}} \right\vert \leqslant \left\vert \sum_{k=1}^{n}\frac{1}{\sqrt{n}} \right\vert = \frac{n}{\sqrt{n}}=\sqrt{n}
$$
所以
$$
P\left( \left\vert \frac{Y_n}{n^{r}} \right\vert >\varepsilon \right) < \frac{\sqrt{n}}{n^{r}\varepsilon}=n^{\frac{1}{2}-r}\varepsilon
$$
$\forall r>\frac{1}{2}$,当$n \to \infty$$n^{\frac{1}{2}-r} \to 0$
所以$\forall \varepsilon>0, \displaystyle \sum_{n=1}^{\infty}P\left( \left\vert \frac{Y_n}{n^{r}} \right\vert >\varepsilon \right) $收敛。
因此$\forall r>\frac{1}{2}$,当$n \to \infty$$\displaystyle \frac{Y_n}{n^{r}}$几乎必然收敛到0。
}
\end{enumerate}
\end{document}